目前分類:多益線上測驗 (18)

瀏覽方式: 標題列表 簡短摘要

Test 12

 

1. The advertising agent, putting up with a low salary for three years, tried to convince his boss that he was worth a ______.

(A) rise (B) raise (C) arouse (D) lift

 

2. Simply let me know ahead of time when you plan to visit us so that I can arrange to have someone ______ then.

(A) capable (B) effective (C) available (D) open

 

3. This whole stretch of road was under the work of resurface ______ heavy traffic has ruined it.

(A) thus (B) because (C) while (D) consequently

 

4. The Crafts Co. decided to offer a bonus to its employees to express its appreciation for their loyalty to the company ______a rough period.

(A) in (B) for (C) at (D) while

 

5. No matter how long it _____to finish a quarterly report, it must be done properly.

(A) taking (B) take (C) takes (D) has taken

 

6. A $50 fee is ______each housing application, but payment of this fee does not guarantee a reservation.

(A) charged for (B) required to (C) reminded of (D) recalled with

 

7. The buyer will need a Bill of Lading (B/L) to collect the goods when they arrive ____their destination.

(A) on (B) at (C) for (D) with

 

8. Many clients have requested that we _____ the products in accordance with the conditions listed on the order.

(A) packed (B) pack (C) packing (D) have packed

 

9 These day-long events, known as quilting bees, were welcome times for getting together with friends and ________community news.

(A) catch on (B) catching at (C) caught up with (D) catching on

 

10. Because of both the opening of new huge new market and major shifts in the workforce, it has become necessary _______ education, training, and experience of employees in the workplace.

(A) supplementing (B) to supplement (C) supplement (D) that supplements

 

11. The Governor of the National Bank is appointed by the President _____the advice of the board of the Economic Promotion Committee.

(A) on (B) with (C) by (D) for

 

12. Mr. Long has just opened a new business and is in the process of ______his first advertising campaign.

(A) launching (B) demonstrating (C) generating (D) performing

 

13. The secretary had found the typographical errors before the messenger ______ the proposal.

(A) delivered (B) had delivered (C) has delivered (D) is delivering

 

14. Massachusetts ranks only 45th in size among the 50 states, but it offers visitors a host of ______ things to see and do.

(A) invited (B) inviting (C) invitation (D) being invited.

 

15. By Friday, seventy-five applicants with a master degree had submitted ____ the teaching position in the university.

(A) at (B) on (C) for (D) by

 

16. An executive _______ innovative ideas from his or her staff.

(A) can sometimes learn

(B) learning sometimes can

(C) sometimes can lean

(D) sometimes can learning

 

17. Professor Lin needs a person to _____ him with the conference display.

(A) assume (B) assign (C) assent (D) assist

 

18. _______you can buy the same brand of soap, toothpaste and sugar-free carbonated drink in all 50 states, the United States reveals major regional differences.

(A) How (B)In spite of (C) Because (D)Although

 

19. If this package is sent by overnight delivery, it _____ New York by noon tomorrow.

(A) reaches (B) will reach (C) is reached (D) has reached

 

20. This is to answer your letter of _______ on the Police Force’s policies and procedures concerning undercover detectives.

(A) require (B) inquire (C) inquiry (D) request

 

21. Dean of the Business Department had students _____ the overhead projector to the conference center.

(A) moved (B) to move (C) moving (D) move

 

22. The _______ rates changed dramatically.

(A) money (B) currency (C) bill (D) coins

 

23. It’s necessary for Ms. Smith to ____ the criteria for recruiting new staff by tonight.

(A) final (B) finalize (C) finally (D) finality

 

24. The variety of insurance benefits _____ very broad under the newly announced policy.

(A) are (B) is (C) have been (D) to be

 

25. Ms Duck was responsible for collecting and ____ the data for the negotiation.

(A) organize (B) organizing (C) organization (D) organized

 

26. The conference site was located in a quiet community; unfortunately, it was much farther than we __________.

(A) have expected

(B) expected

(C) had expected

(D) was expected.

 

27. I suggested ______ in more exhibitions to make our product well-known to the public.

(A) participate (B) to participate (C) participating (D) participated.

 

28. The host has no idea how much food to prepare for guests who ______the wedding banquet.

(A) are attending (B) attending (C) attended (D) they will attend

 

29. Mr. Welsy ______ his schedule by himself.

(A) always arranges

(B) arranges always

(C) has arranged always

(D) always to arrange

 

30. _____all the sales representatives, Mr. Teller has been the most energetic and intelligent.

(A) For (B) As (C) Of (D) With

 

31. Mr. Lee is a man with sound financial situation; ________ he never borrows money or asks for loans from the bank.

(A) in spite of (B) in fact (C) on the other hand (D) while

 

32. An terrible earthquake occurred _____ the doctors were conducting a medical operation.

(A) so (B) while (C) because (D) nevertheless

 

33. Roma’s historic places, architectural highlights, and overall charm are ________by a rich cultural heritage.

(A) complementing

(B) complemented

(C) to be complemented

(D) complement

 

34. Since Peter enrolled in the EMBA program, he has _____ a knowledgeable trainee.

(A) looked into

(B) brought about

(C) turned into

(D) broken into

 

35. While the presentation _______, the technician handled the computer displays.

(A) was being made

(B) has been made

(C) was making

(D) made

 

36. The products need to be tested for their safety _____ durability.

(A) but (B) as well as (C) or (D) yet

 

37. Ms. Wu was hardworking and competent; ____ she had no trouble being recommended.

(A) namely (B) nevertheless (C) moreover (D) thus

 

38. The words, FRAGILE – Handle with Care, are ______on all sides of the cases, which are individually numbered from 1 to 10.

(A) stenciled (B) stenciling (C) steered (D) stressed

 

39. If we ____ more aware of the trends, we could have avoided such a great loss.

(A) are (B) were (C) have been (D) had been

 

 

解答                                                               

1-10

A C B B C

A B B D B

11-20

B A A B C

A D D B D

21-30

D B B B B

C C A A C

31-39

B B B C A

B D A D

 

 

 

Zeus 發表在 痞客邦 留言(0) 人氣()

Test 11

 

1. The university alumni include a _______ group of engineers, architects, technicians, business executives, and others holding key positions throughout Taiwan.

(A) distinct

(B) distinguish

(C) distinguished

(D) distinctively

 

2. Prior to joining our company, Dr. Shwartz was an ______ professor of economics for more than a decade.

(A) winning-award

(B) win-award

(C) award-winning

(D) award-winner

 

3. Ms. Kitagawa spoke quite excitedly while she _______her sales presentation.

(A) took

(B) was taking

(C) has taken

(D) had taken

 

4. The faculty and staff get their paychecks automatically _____ in their bank accounts.

(A) deposited

(B) depositing

(C) are deposited

(D) deposit

 

5. The client has ______ a lawyer to act for her while she is abroad.

(A) authority

(B) authorized

(C) authorization

(D) author

 

6. To respect the rights of non- smoking persons and in accordance with state regulations, smoking is _______in all public buildings.

(A) prohibited

(B) proposed

(C) propelled

(D) prospected

 

7. Using the computer to distribute unwanted messages or ______other users may result in disciplinary actions.

(A) disturbing

(B) to disturb

(C) by disturbing

(D) disturbance

 

8. Housing reservations are made ____ a first come, first served basis in the sequence in which completed application forms are received.

(A) on

(B) with

(C) for

(D) above

 

9. The great deciduous forest in New England presents a display of colorful fall foliage, which is unequaled ______ in the world.

(A) nowhere

(B) wherever

(C) anywhere

(D) where

 

10. ________ there has been a strong competition, the new company has made great profits.

(A) Instead

(B) Nonetheless

(C) Because

(D) Although

 

11. It’s the policy of the company to maintain a work environment free of sexual harassment, ______equal employment opportunities.

(A) it impedes

(B) impedes

(C) which impeding

(D) which impedes

 

12. The ______ regarding whether Cape Cod looks like a flexed arm or a fishhook, has never been settled.

(A) dispute

(B) disputing

(C) disputable

(D) disputant

 

13. An increase in the cost of raw materials –____50% so far – will inevitably lead to price increase for our customers.

(A) up to

(B) equal to

(C) go to

(D) run into

 

14. All students may drive motor vehicles on campus; _____, parking is by permit only.

(A) but

(B) while

(C) thus

(D) however

 

15. The electricity company is not liable for damage caused by natural disasters _______ its control.

(A) beyond

(B) above

(C) outside

(D) around

 

16. On international shipments, all duties and taxes are paid by the ________.

(A) recipient

(B) receiving

(C) receipt

(D) receptive

 

17. Because of the _____ weather conditions, Taiwan has an advantage in the production of fruits and the promotion of touring business.

(A) favorite

(B) favor

(C) favorable

(D) favorably

 

18. The _______ concerning the working hours between labors and employers will be held at the International Trade Center.

(A) negotiations

(B) dedications

(C) propositions

(D) appreciation

 

19. In order to make more money, Mr. Clinton decided to ____ a second job.

(A) take off

(B) take out

(C) take from

(D) take on

 

20. The company has to list its assets ____ liabilities at the stockholders meeting held annually in late June.

(A) but

(B) nor

(C) or

(D) and

 

21. Anyone who is interested in the position needs to ____ his or her resume to the personnel director prior to the application deadline.

(A) request

(B) adopt

(C) submit

(D) substantiate

 

22. _____ has the interest of time deposit been much lower and subject to change.

(A) Negatively

(B) Rarely

(C) Consequently

(D) Regularly

 

23. Have the secretary _____ my office before she leaves today.

(A) stop over

(B) stop for

(C) stop by

(D) stop to

 

24. Regulations must be examined and discussed _____ before they are announced.

(A) thoroughness

(B) thorough

(C) thoroughly

(D) more thorough

 

25. Engineers must be both _______ and creative and must exercise leadership to accomplish goals.

(A) analytical

(B) analyze

(C) analyst

(D) analysis




 

Zeus 發表在 痞客邦 留言(0) 人氣()

Test 10

 

1. The purchaser suggested that the equipment ______ by next Monday morning.

(A)delivered

(B)delivering

(C)be delivered

(D)was delivered

 

2. The benefits packages offered by the Central Insurance Co. is impressive, ____ it is promoted only from within the company.

 

(A)for example

(B)when

(C)but

(D)nevertheless

 

3. ______the superior quality of our designs and craftsmanship, we cordially invite you to our special exhibition at the King Hotel.

 

(A)Full appreciation

(B)To fully appreciate

(C)To full appreciating

(D)Fully appreciate

 

4. Ten years ago, there _____ a severe shortage of computer specialists.

 

(A)was

(B)were

(C)has been

(D)have been

 

5. The need for in-service training programs ________ as a result of advances in global competition, consumer expectations, and the opening of huge new markets.

 

(A)is creasing

(B)are increased

(C)have increased

(D)increased

 

6.The 200-page, fully-illustrated _____ contains full details of our complete range of products.

 

(A)leaflet

(B)brochure

(C)catalogue

(D)booklet.

 

7. We would appreciate it if you would _______ these goods to be dispatched together with the same terms as the original order.

 

(A)confine

(B)confirm

(C)coincide

(D)conduct

 

8. The Bank of England ______in 1694 by a group of wealthy merchants and landowners for the purpose of raising a loan for King William III.

 

(A)was founded

(B)was found

(C)found

()has been founded

 

9. When considering if an applicant is suitable for a job, we will compare his or her previous experience and professional training with the _______of the job.

 

(A) requests

(B) reforms

(C) restrictions

(D) requirements

 

10. A smile, a gesture with the hand or a meaningful standing position ____ a neglected area of the salesman’s skills.

 

(A)is

(B)are

(C)was

(D)to be

 

11. After having _____ the matter, we discovered some confusion in the numbering of two different orders, which were collected for dispatch to Canada.

 

(A) looked up

(B) looked into

(C) looked over

(D) looked for 

 

12. The firm decided to take further action to make the unpaid invoice of $90,000 payable, which has caused ______financial difficulties.

 

(A)considerate

(B)considering

(C)considerable

(D)considerably

 

13. Mr. Lopes has ______ learned to send emails and to copy diskettes.

 

(A) yet

(B) soon

(C) ever

(D) already 

 

14. Many conference attendees were late because the hotel was located farther than anyone __________.

 

(A) anticipated

(B) have anticipated

(C) was anticipated

(D) had anticipated

 

15. It was predicted that an _______ depreciating Taiwan dollars would stimulate exporting business in Taiwan.

 

(A)increased

(B)increasing

(C)increase

(D)increasingly

 

16. Good prospects for the position should be well-motivated persons with an initiating and flexible attitude to a variety of job.
            A                                                                        B                                      C                                                                            D

 

17. Because of the satisfaction of our customers is our primeconcern, we are contacting all the owners of this year’s March to alert them to

              A                                                                                                                                                                                                           B                               

      improvements in the design of one of the car’s features.
                            C                                                  
D

 

18. Ms. Howard who spent several years at our offices in Zurich, she shared with us her ideas about contemporary patterns in trade.

                                    A                                   B                                      C                                                                                                 D

 

19. The number of aeronautical engineers required meeting air  transportation needs is rapidly increasing.
            A                                                                B                   C                                                                 D

 

20. The credit cards issued by Jade Bank will bring you benefits that provides greater financial flexibility.
                                    A                                       B                           
      C                                       D

 

21. When she liaising with committees and senior management,Ms. Lee is able to work under pressure and organize and to communicate effectively.
                          A                                                                                                                        B                                                                C

 

22. As a young man, Alexander Graham Bell had dedicated his life to help deaf people, trying to make extra money by working out how to improve

                                                                                                                      A                                                           B                            C                          D

     the telegraph.
                  

23. Commercial cycles have been placed the scrutiny of statisticians so that recurring patterns not obvious at first sight might be disclosed.
                                              A                                                                      B                                                             C

 

24. Fish have a well-developed sense of hearing, and the ability of transmitting sounds. 
             A                B                                 C                                   D

 

25. Few blankets made by native Taiwanese would have survived to the present were it not due to enthusiastic collectors at the turn of the century.
   A                                                                                                                                   B               C                                            D

 

26. With the increase of automation, the problems of unemployment will become more serious unless more people would be given the training

          A                                                                                                                                      B                C                                            D    

     necessary for white collar positions.
             

 

27. Unlimited liability means that if a business gets into debt and eventually fails, then all the private wealth of the owner can be used to pay those

                                                                                               A                                                                       B                                                                          C

     people the business owns money to. 
                                 D

 

28. More than two people who contribute an amount of money for investment through a partnership will share the profit upon agree proportions .
                                                                              A                                                                                             B                                     
   C

 

29. During the spring runoff, streams and rivers run full and waterfalls are with their most colorful.
       A                                                                          B          
                         C    D

 

30. The association holds meetings and sponsors a widely variety of activities and services for all the medical graduates in Taiwan. 
                                 A                                                        B            
                                                                                                        C        D

 

31. Counseling in resume writing, interviewing and job search  strategies are offered.
               A                                                   B                             C        
                    D

 

32. The staff of professional librarians, in duty during all scheduled hours, are skilled in locating information. 
                                                                       A                                B       
            C                    D

 

33. Continue education and professional development are vital components in career growth.
              A                                                                                  B    C                                            D

 

34. Roadside stands offer local honey and a unbelievable range of shapes and sizes of fat pumpkins. 
             A                         B                               C                                      
    D

 

35. Total amount of the membership dues collected at the annual conference was more than $40,000. 
                    A                                               B         C                                                                 D




 

Zeus 發表在 痞客邦 留言(0) 人氣()

聚會是英國人最熱衷的社交活動之一,它分為不同的類型,比如烤肉派對、生日宴會、聖誕聚餐、單身派對、訂婚宴會等。你會正確使用聚會時常能聽到、用到的英語俚語和慣用詞彙嗎?做測驗試試看吧~

 

多益線上測驗-聚會.jpg

 

1. I'm going to _________ a party to celebrate my 21st birthday.

(A) do

(B) make

(C) catch

(D) throw


2. Which is a slang term for having a party?

(A) She's having a bash at her place tonight.

(B) She's having a smash at her place tonight.

(C) She's having a flash at her place tonight.

(D) She's having a rash at her place tonight.


3. In which sentence does the hostess offer to provide small snacks?

(A) Please come to my party. I'll sort out the nibbles but can you all bring a bottle.

(B) Please come to my party. I'll sort out the party poppers but can you all bring a bottle.

(C) Please come to my party. I'll sort out the costumes but can you all bring a bottle.

(D) Please come to my party. I'll sort out the Christmas crackers but can you all bring a bottle.


4. He was _________ and talked to everyone and encouraged them to have a good time.

(A) the stag

(B) the life and soul of the party

(C) party pooper

(D) hostess

 

5. What is the name of a party that an unmarried woman has to celebrate her upcoming wedding?

(A) hen night

(B) stag night

(C) baby shower

(D) reception

 

6. We all had a _________ of a time at the party. Everyone really had a lot of fun.

(A) bear

(B) lion

(C) whale

(D) tiger

 

 

答案

1.(D) 2.(A) 3.(A) 4.(B) 5.(A) 6.(C) 


多益線上測驗-電話用語.png


我們在打電話時會用到一些專門的單詞和搭配,比如,詞組hold on 的意思是讓對方等一下,先別掛電話;而詞組hang up 是“掛電話、掛斷”的意思。與人通話時,還要分清並掌握正式和非正式說法的區別。來試試下面幾道英語測試題吧!


1. I already owned a mobile phone so when I arrived in the UK I got a _________ so I didn't have to pay for a handset.

(A) smartphone

(B) mobile phone bundle

(C) SIM-only contract

(D) tablet


2. Which of the following is an INFORMAL way to request call again later?

(A) Give us a bell later and we'll have a chat.

(B) Could you call me back this evening?

(C) Please ring me back during office hours and we can continue our discussion.

(D) Please hang up and redial the number required.

 

3. Which sentence requires the caller to wait to be attended to?

(A) Give us a buzz later.

(B) Could you hold the line please?

(C) I have a call on another line.

(D) Can you call me back later?

 

4. I can't speak now. I'll _________ in five minutes.

(A) call you forward

(B) call you over

(C) call you round

(D) call you back

 

5. Which sentence requires the caller to wait to be connected?

(A) Hold on. I'll pull you through.

(B) Hold on. I'll carry you through.

(C) Hold on. I'll put you through.

(D) Hold on. I'll follow you through.


6. Sorry the reception's bad on my mobile. Can you call my _________?

(A) landline

(B) wired phone

(C) non-mobile line

(D) fax machine

 

 

答案

1.(C) 2.(A) 3.(B) 4.(D) 5.(C) 6.(A)
 

Zeus 發表在 痞客邦 留言(0) 人氣()


7fe7b478-dfb3-4ec6-b868-db7413a4ea4f.jpg_wh1200.jpg
 

學生,做作業可是頭等要事兒,不過你分得清各種作業,比如dissertation 論文和essay 文章的區別嗎?還有三明治課程究竟是什麼意思?本期《英語小測驗》挑戰你對大學裡一些常用表達的理解。

1. Students are reminded that all coursework must _________ by the end of May.

(A) be handed in

(B) be handed

(C) hand in

(D) hand out

 

2. I had to write a 25,000-word _________ to complete my Master’s degree.

(A) email

(B) report

(C) dissertation

(D) presentation

 

3. What is ‘Freshers’ Week’?

(A) A week of celebration following graduation.

(B) An introductory week of activities for first-year students joining the university.

(C) A week of preparation before the first-year exams.

(D) A week of interviews to select new students for a university.

 

4. What is a ‘sandwich course’?

(A) A course at a culinary college in food preparation.

(B) A part-time course in which students study in their lunch hours.

(C) A special course that takes place during the university summer holidays.

(D) A university course in which students also take on full-time work related to their field of study.

 

5. What description best characterises a seminar?

(A) A speech delivered by a tutor to a large group of students.

(B) A one-to-one session between a teacher and a student.

(C) A class at university in which a particular topic is discussed by a tutor and a small group of students.

(D) A series of tasks that students must complete over the course of an academic year.

 

6. He was a first-rate student and won a _________ to study at Oxford University because of his academic excellence.

(A) scholarship

(B) grant

(C) bursary

(D) student loan 

 

答案

1.(A) 2.(C) 3.(B) 4.(D) 5.(C) 6.(A)

 

英文00.jpg
 

我們在日常辦公時需要處理各種各樣的事情和應對突發情況,比如組織電話會議,解決電腦、打印機小故障,群發郵件等等。本期《英語小測驗》測試你對辦公室裡常用語的掌握,來試試你知道多少個。


1. There is a meeting at 10.30 where we will have _________ with our counterparts in Europe and the Middle East.

(A) a conference call

(B) an instant message

(C) cosy chat

(D) launch party

 

2. Can someone call _________? The email network has gone down again.

(A) lab technicians

(B) marketing team

(C) sales department

(D) IT support

 

3. Oh no, the printer has run out. We need to order some more _________.

(A) paint

(B) paper clips

(C) ink cartridges

(D) hole-punches

 

4. My American colleague refers to office gossip as _________.

(A) an appraisal

(B) water-cooler chat

(C) an interview

(D) a pitch

 

5. I hate business jargon. Why do people say “think _________” instead of thing originally?

(A) inside the box

(B) outside the box

(C) over the box

(D) behind the box

 

6. My boss always asks me to _________ emails down instead of just saying pass this email on to the others.

(A) flow

(B) run

(C) cascade

(D) despatch

 

 

答案

1.(A) 2.(D) 3.(C) 4.(B) 5.(B) 6.(C) 

 

Zeus 發表在 痞客邦 留言(0) 人氣()

多益線上測驗-英式和美式.jpg

英式英語和美式英語:相同表達中措詞的區別

具有相同含義的英語表達在英式和美式英語中措詞可能有所不同。比如,“像家裡一樣舒適的地方”的英式表達是a home from home,而美式英語裡寫成a home away from home;當英國人祈求好運時說“touch wood 摸木頭”,而美國人則說“knock on wood 敲木頭”。兩種英語分支同大於異,然而為避免誤會,最好分清二者的區別。試做六道小題,學習英式英語和美式英語的同義表達中措詞的差別。

1. An American English speaker would use the expression 'I could care less.' What would a British English speaker say?

(A) I don't care less.

(B) I won't care less.

(C) I shan't care less.

(D) I couldn't care less.

 

2. A British English speaker would say, 'Could you please hold the fort?' What would an American English speaker say?

(A) Could you please hold up the fort?

(B) Could you please hold around the fort?

(C) Could you please hold down the fort?

(D) Could you please hold in the fort?

 

3. An American English speaker would say, 'Take it with a grain of salt.' What would a British English speaker say?

(A) Take it with a knob of salt.

(B) Take it with a pinch of salt.

(C) Take it with a nip of salt.

(D) Take it with a grit of salt.

 

4. An American English speaker would say, 'I'm sorry. My bad.' What would a British English speaker say?

(A) I'm sorry. My worse.

(B) I'm sorry. My mistake.

(C) I'm sorry. My evil.

(D) I'm sorry. My atrocious.


多益線上測驗-英式和美式01.jpg

 

5. A British English speaker would say, 'That's thrown a spanner in the works.' What would an American English speaker say?

(A) That's thrown a wrench in the works.

(B) That's thrown a screwdriver in the works.

(C) That's thrown a hammer in the works.

(D) That's thrown a jackhammer in the works.

 

6. An American English speaker would say, 'Don't sweep it under the rug.' What would a British English speaker say?

(A) Don't sweep it under the floorboards.

(B) Don't sweep it under the floor towel.

(C) Don't sweep it under the mat.

(D) Don't sweep it under the carpet.

 

答案

1.(D) 2.(C) 3.(B) 4.(B) 5.(A) 6.(D)


多益線上測驗-英式和美式02.jpg

英式英語與美式英語:語法的區別


雖然英式英語和美式英語的詞彙基本一樣,但是在語法上它們還是有細微的區別的。比如,在問別人“你有筆嗎”時,英國人是會說“Have you got a pen?”還是“Do you have a pen?”。試做《英語小測驗》,豐富你的英語知識。

1. An American English speaker says 'I already did it' to you. What might a British English speaker say instead?

(A) I already done it.

(B) I've already done it.

(C) I'd already done it.

(D) I already did it?

 

2. How does British English compare to American English in regards to collective nouns? E.g. team, group, family, jury etc.

(A) Both British and American only use singular verbs following a collective noun.

E.g. The team is losing.

(B) British English uses a plural verb following a collective noun, whereas American English will only use a singular verb. E.g.:

British: The team are losing.

American: The team is losing.

(C) British English uses a singular verb following a collective noun, whereas American English will only use a plural verb. E.g.:

British: The team is losing.

American: The team are losing.

(D) British English uses either a singular or plural verb following a collective noun, whereas American English will only use a singular verb. E.g.:

British: The team is / are losing.

American: The team is losing.

 

3. How do British and American English treat the verbs 'dream', 'burn', 'smell', and 'learn' among others in regards to their past simple and past participle forms?

(A) There is no difference between American and British English. These verbs are conjugated with ed endings: dreamed, burned, smelled, learned.

(B) There is no difference between American and British English. These verbs are conjugated with t endings: dreamt, burnt, smelt, learnt.

(C) American English prefers the-ed ending: dreamed, burned, smelled, learned. But, British English can use the t ending: dreamt, burnt, smelt, learnt.

(D) American English prefers the-t ending: dreamt, burnt, smelt, learnt. But, British English can use the ed ending: dreamed, burned, smelled, learned.

 

4. An American English speaker would ask 'Do you have a pen?' How could a British English speaker ask also using the verb 'have'?

(A) Do you have a pen?

(B) Do you have a pen?

Have you got a pen?

(C) Do you have a pen?

Are you having a pen?

(D) Have you gotten a pen?

Are you having a pen?

 

5. In a formal situation, how might an American English speaker and a British English speaker talk about calling someone in the future?

(A) American English: I will call you early tomorrow morning.

British English: I shall call you early tomorrow morning.

(B) American English: I will call you early tomorrow morning.

British English: Im gonna call you early tomorrow morning.

(C) American English: I will call you early tomorrow morning.

British English: I must call you early tomorrow morning.

(D) American English: I will call you early tomorrow morning.

British English: I call you early tomorrow morning.

 

6. In British English the verb 'get' is conjugated 'get / got /got'. E.g. I get money. I got money. I've got money. How is this done in American English?

(A) It is the same: get, got, got.

(B) American English uses: get, got, gotten.

(C) American English uses: get, gotten, got.

(D) American English uses: get, getten, gotten.

 

 

答案

1.(B) 2.(D) 3. (C) 4.(B) 5.(A) 6.(D)

 

多益線上測驗-英式和美式03.jpg

 

Zeus 發表在 痞客邦 留言(0) 人氣()

Test 10

 

1.Men often wait longer to get help for medical problems than women, and         , women live about six years longer than men.
(A) instead of
(B) constantly
(C) consequently
(D) because


2.           she manages three companies, Sara still has time to coach her son’s soccer team.
(A) Because of
(B) Weather
(C) Often
(D) Although


3.Tom played basketball in college and          active ever since.
(A) have extremely been
(B) has been extremely
(C) will be extremely
(D) should extremely be


4.The guests at  my fantasy  dinner party          Abraham Lincoln, Marco Polo, St. Peter, and Socrates.
(A) will be included
(B) should have invited
(C) would include
(D) are including


5.When I             your  friend  Jamie this Sunday,  I  will certainly give him your message.
(A) have seen
(B) will see
(C) see
(D) sees


6.All  of  the  international  dignitaries  attending the conference to                  bring a souvenir from their country.
(A) has asked
(B) asking
(C) were asked
(D) was asking


7.My grandmother  has  been going  to a  better dentist,   so this the problems she is having                with her dentures.
(A) won’t eliminate
(B) will be elimination
(C) should have been eliminated
(D) should help eliminate


8.After  my  Aunt  Sheila  turned 65, she          to  celebrate  life more by going to the ballet and the opera.
(A) decided
(B) will decide
(C) have decided
(D) will be deciding


9.Our  new  firm             for  a  credible,  aggressive  individual  with great communication skills to fill this position.
(A) have looked
(B) are looking
(C) is looking
(D) look


10. I didn’t know           of my hair he was going to cut.
(A)how
(B)how small
(C)how many
(D)how much


11. If you want to do well on the exam, you             on the directions that the professor gives and take exact notes.
(A) will have concentrated
(B) have to concentrate
(C) will be concentrated
(D) will be concentrating


12.            the  heavy  pollution,  the  city  officials  have  decided to cancel school for the day.
(A) Prior
(B) By means of
(C) Due to
(D) Though


13. Unless there is a blizzard tonight,  the snow       deep enough for the skiing competition tomorrow.
(A) won’t be
(B) won’t have
(C) wouldn’t have had
(D) isn’t


14. Terry’s mother kept telling him that         in the street is dangerous, but he would not listen..
(A) played
(B) will play
(C) playing
(D) been playing


15. When you leave, make certain all the windows and doors         .
(A) will close
(B) should have bben closed
(C) are closed
(D) is closed


16. He wouldn’t even think  of wearing         clothes; they make him look so old!
(A) same
(B) despite
(C) such
(D) that


17. After the performance of Othello,  the audience         to their feet and gave a thunderous standing ovation.
(A) have rose
(B) rose
(C) risen
(D) rising


18.         arose among all of the students as to why the teacher was fired.
(A) Suspicious
(B) Suspect
(C) Suspicion
(D) Suspecting


19. When Jessica’s brother hit her with a stick,   she              cried out for her mother.
(A) quick
(B) immediate
(C) instantaneous
(D) immediately


20. Michael Jordan made  the most           three-pointer at the sound  of the buzzer and won the championship for his team.
(A) humdrum
(B) unpretentious
(C) ridiculous
(D) spectacular


21. Our dog Bubbles just gave birth to six black-and-white puppies but we can’t get close to see them because she is                         of them.
(A) very private
(B) very protective
(C) too professional
(D) too precise


22. The  advertisement  for  Super Suds detergent             that  the sale  has increased by 25% in the first quarter.
(A) have been so successful
(B) had been so successful
(C) has been so successful
(D) will be so successful


23. Tom Jones, who             around the world, will come to Asia next month.
(A) will be touring
(B) have toured
(C) had been touring
(D) has been touring


24. That sailing ship has just taken a sharp turn and  it looks          it were going to turn over.
(A) as through
(B) although
(C) as though
(D) as thorough as


25. My airline reservations have been changed so many times that I am totally                     as to when I need to be at the airport.
(A) confident
(B) bewildered
(C) confusing
(D) complicated


26. Our boss taking everyone to the ballet tonight, and I need to make sure my new dress   for the occasion.
(A) has been cleaned
(B) should have been cleaned
(C) is being cleaned
(D) has been cleaning


27. Lucinda felt that she wasn’t sharp enough         as businesswoman.
(A) for being
(B) to being
(C) to be
(D) as to be


28. After all the late night studying, Janet  was too       to stay awake for her last exam.
(A) excited
(B) exhausted
(C) exhumed
(D) exhilarated


29. The  instructions on how  to  use the  new machine            that nobody seemed to be able to understand.
(A) were very simplistic
(B) was very confused
(C) were so confusing
(D) was so simplistic


30. Our  country  hasn’t seen an           problem  for  years  because of the strong economy.
(A) unemployed
(B) unemployment
(C) unemployer
(D) unemploying


31. This music group’s popularity has         over the past five years.
(A) growing
(B) grown
(C) been grown
(D) grew


32. The professor gave us          different solutions to this science question.
(A) as many
(B) few
(C) a few
(D) much


33. Many of the fads of the 1970’s          as today’s latest fashion.
(A) are being revived
(B) is revised
(C) are revoked
(D) is being reviled


34. Sally doesn’t like that pair of shoes because they are too           .
(A) loosen
(B) lose
(C) loose
(D) loss


35. My mother always prepared each course of the meal very        .
(A) careful
(B) careless
(C) carefully
(D) caring


36. Our car trunk           with suitcases and we could hardly make room for anything else.
(A) went cramming
(B) was crammed
(C) is cramming
(D) was been crammed


37. Sally  me  use  her  car  to  drive  to  the grocery store          buy some snacks for the party this weekend.
(A) due to
(B) because
(C) instead of
(D) in order to


38. “The titanic” was such a popular movie that it seemed to          the whole world.
(A) almost keep
(B) practically shout
(C) literally sweep
(D) somewhat keep


39. Sally found a rare kind of fossil         down this path.
(A) furthest
(B) farther
(C) farer
(D) more far


40. He  put  the history books          the  dictionaries  and other reference book on the shelf.
(A) beside
(B) next
(C) besides
(D) adjacent

 

 

 

Answer

 

1.      C

2      D

3      B

4      C

5        C

6      C

7      D

8      A

9      C

10      D

11    B

12    C

13    A

14    C

15      C

16    C

17    B

18    C

19    D

20      D

21    B

22    C

23    D

24    C

25      B

26    A

27    C

28    B

29    C

30      B

31    B

32    C

33    A

34    C

35      C

36    B

37    D

38    C

39    B

40      A

 

Zeus 發表在 痞客邦 留言(0) 人氣()

 

多益考試決戰日期近在眼前,手邊想找題目或來練練自己的實力或者還在擔憂自己的英文實力能不能去應試嗎?不用擔心,以下幾個網站皆是免費的多益測驗試題裡面有多元的題目、各式各樣的難度,無論是想簡單程度或初學程度,還是本身想多挑戰高難度的測驗,歡迎大家來使用這些資源,來檢測本身有多少實力吧,若發現程度有待加強,剛好再努力做最後的衝刺!為自己的英文能力、未來多多加油!

 

TOEIC-09.jpg

 

聽力部分

 

 

http://www.esl-lab.com/


提供非常多的簡短對話練習,並且依難易度區分,對於不同程度的朋友刀能找到適合自己的節奏,除了聽力問答之外,每則測驗還包含了重點字彙、片語等等的額外練習,在準備考證照或英聽的同時也能擴充自己本身的單字量。

 

http://www.english-test.net/toeic/listening/autobahn.html


模擬整個多益的聽力部分,圖片敘述、問題回應到簡短會話都有龐大的題庫可以練習。每則練習都可以將讀稿顯示出來,清楚檢視聽不懂的地方。整體來說難易度接近,但稍異於實際多變。

 

TOEIC-14.jpg

 

https://tw.voicetube.com/


搜集豐富主題的英文短片,在獲取新知識的同時又能增長英聽能力。站中同時將各個影片標上難易度以及講者所屬腔調的標籤 (如美國腔、英國腔、澳洲腔等等),對於廣納各國腔調的多益測驗來說是非常好的練習。

 

https://www.facebook.com/permalink.php?id=151750254989692&story_fbid=208587809305936


倘若你正缺考題練習的話,這裡有考題大放送讓你在考試準備期間,不用擔心沒有試題可以檢測你的英文準備狀況,還可以從考題當中學習文法、單字、片語,讓你不用花大錢就可以有高分英文證照!
Youtube影片(只有聽力部分試題 解答在影片說明那邊)

 

多益-6.jpg

https://www.youtube.com/watch?v=YBQ-irnWrmY&list=RDYBQ-irnWrmY#t=9


這是多益聽力試題合輯版本,有多樣及大量的多益聽力試題,然後解答就寫在下方,有差不多5~10課左右,看一下上面的評論寫說,基本難度偏高。如果不介意的同學可以前往參考,也可以測試自己的程度!剛好可以替自己省一點荷包喔!還在找聽力的同學歡迎來使用這個免費網站喔!

 

https://www.youtube.com/watch?v=7ztmYPZTtlc


再推薦一個美國脫口秀的節目吧,想必大家做了這麼長一段的考題身心都累了吧,看看節目休息休息。剛好也可以練練各腔調的英文。
 


以上的網站是根據網友們的真心介紹推薦,雖說稱不上很多,但是裡面的英文知識還等著你我來學會,剛好以上的網站,是不用花什麼錢去買聽力參考書的,來多多把握!不管哪一種學習方式,只要你肯靜下心,用心學習並努力檢討過錯,勇往直前,一定會有佳績!


最後預祝各位大家可以奪得心目中理想佳績,拿到漂亮分數的證照!

 

TOEIC-04.jpg
 

Zeus 發表在 痞客邦 留言(0) 人氣()

比較結構

 

比較結構藉著形容詞副詞的字形變化來比較兩個人、事、物。

 

壹、One comparative criterion

 

 一、原級比較

比較對象

動詞

Adv.

形容/副詞

連接詞

被比較對象

  1. She

is

as

charming

as

her sister is.

  1. I

drive

as

carefully

as

 

  1. He

earns

as

________ money

as

he does.

  1. He

read

as

________ books

                 as

I do.

  1. He

earns   

____ as

 much money

                 as

I did. 

I do.


二、比較級

比較對象

動詞(受詞)

形容/副詞 + er

連接詞

被比較對象

  1. Ann

worked

harder

than

he _______.

  1. He

speaks

more fluently

than

______I

  1. John

knows food

better

than

Jane knows wine.

 

  • 同樣事物才能比較: My car is more expensive than _________________.
  • 避免雙重比較的錯誤:  * She is more taller than her brother.
  • 慣用語: higher education, rather than, more than (=over)
  • 拉丁語系: 本身已有比較的意味, 不須加’er’或’more’, 以’to’代替’than’

He is ________ superior _______ me in English writing.

(= He is ______ better than ______  am in English writing.)

  • 有些形容詞沒有比較級: unique、perfect、 single、 round、 square。

 

三、最高級

 

補語/副詞

Christ

 is

 the tallest (girl)

in class/ among these girls.

Einstein

 was

 the greatest (scientist)

 

He

 is

 the best speaker

of all scientists

He 

 speaks

 best

in the contest.

In the contest.

 

貳、More than one comparative criterion

雙層比較結構 【the more + adj/adv + S +V, the more adj/adv + S1 + V1

When you drive faster, you will feel better.= The faster you drive (因), the better you feel.(果)

  • The older (one becomes), the wiser (one becomes).
  • The more expensive the hotel (is), the better the service (is), which is offered.

 

題目來囉~~

 

1. The more equipment activities require,       .

 

(A)which the activities become less accessible

(B)and the less accessible activities become

(C)the less accessible the activities become

(D)activities become less accessible

 

2. The stronger         magnetic field, the greater the voltage produced by a generator.

 

(A)than the          (B) is the   (C) that the          (D)the

 

3. The higher the standard of living and the greater the national wealth, the      .

 

(A)greater is the amount of paper is used

(B)greater amount of paper is used

(C)amount of paper is used is greater

(D)greater the amount of paper used

 

4. Grand Canyon itself defies human comprehension. The more you know about it,           .

 

(A)the more awesome it becomes

(B) it becomes more awesome

(C) than it becomes awesome            

(D) and more awesome it become

 

5. The annual worth of Korea’s manufacturing is greater than       .

 

(A)that of its mining and farming combined

(B)mining and farming combination

(C)that mining and farming combined

(D)of its combination mining and farming

 

 

Zeus 發表在 痞客邦 留言(0) 人氣()

Test 9

 

1.____ interest rates will add to the costs because the clients have to pay more money to the bank from which they borrow the money.

 

(A) Rising (B) Rises (C) Raising (D) Rise

 

2. Government imposes many regulations on foreign business, from consumer rights, to employee and labor relations, _____ to laws governing contracts and liability.

 

(A) or (B) and (C) either (D) but

 

3. P&O is always looking to get the best value for money; _____ it is also interested in the quality of the product, delivery times and after sales service.

 

(A) although (B) in spite (C) however (D) accordingly

 

4. ________the difficulty of using different currencies, the EU has established a monetary union where there is only one single currency, the Ecu.

(A) To get around

(B) Getting up

(C) With getting hold

(D) To get through

 

5. Beardshaw’s did very well _____ the economy was booming, but when a recession came, it saw sales fall dramatically.

 

(A) during (B) for (C) while (C) when

 

6. A multinational needs to develop strategies for _____ a number of different problems, such as communication, local laws and politics, and exchange rate fluctuations.

 

(A) bringing up with

(B) coping with

(C) dealing in

(D) setting up

 

7.Six countries account for 80 percent of McDonald’s foreign ______. So there is plenty of room for expansions worldwide.

 

(A) earns (B) earning (C) earned (D) earnings.

 

8. In women’s eyes, public achievement makes a man more attractive as a marriage partner but for men the situation is______.

 

(A) reversed (B) reserved (C) replaced (D) recognized.

 

9. Many companies set a number of objectives, ________ is an indicator of how well the company is performing.

 

(A) each (B) which (D) each of which (D) it

 

10. Studies showed that the accumulation of carbon dioxide ______ average global temperatures to rise nearly four degrees Fahrenheit by the twenty–first century.

 

(A) have caused (B) has caused (C) have been causing (D) causing

 

11. Gambling is a controversial subject, which leads to debate and discussions.
         A                       B                                            C        D

 

12. According to the law, every partner gets an equal share of the profit and has the same voting power as any other partners.
                                                                               A                                       B                                   C                                     D

 

13. Farmers in the developed countries of the world is coming  under increasing pressure from ecological groups to reduce their use of pesticides

                                                                                                 A                                   B                                                                              C  

       and fertilizers. 

                 D                                                                               

                           

                                            

 

14. If the interest rate was to go up, the interest charge in dollars would be even higher.
                                   A                                                          B     
          C                  D

15. The district has one of the largest attraction in Taiwan’s tourist region and has an increasing number of “highly commended” properties in the

                                                                                           A                                                                                                        B                                   C

      accommodation list.
              D

 

16. Businesses are constantly having to make cost decisions. For the same price, what would have to be given up if a computer is bought?
                      A          B                                               C                                                                                           D

 

17. The director of the publication department works harder as his staff do.                            

                                               A                                     B                C                  D

 

18. It is ridiculous for people in the same office to use e-mail to communicate with the one another.
 A                                                                              B                       C                                 
   D

 

19. That building, its architect is unknown, is a popular tourist attraction.
                           A                  B                   C                         
        D

 

20. Men are typical motivated by a social structure that says if you don’t dominate, you will be dominated.
                     A                                                                B                                                                     C

 

21. The cash, which it was a bonus for being number one, totaled more than theprevious bonuses put together.
       A                 B                                                  C                    
                                                                                 D

 

22. In order to survive the competition of other firms, successful businesses have to supply goods and services that customers want to buy, with the

                                                                       A                                                                                                                          B         C                                               D

      right price. 
                                                   

                                                                   


                             

23. In a free market economy, companies are private businesses  that are not owed by the state.
                                     A                                             B                           C             D

 

24. Machines have replaced workers and thus a few factories today employ mostly no labor at all.
                            A                                 B                                                            C                        D 

 

25. Convincing evidence from the field of behavioral genetics implies that certain biological predispositions to criminal behavior is inherited.
         A                                                                                          B                                                          C             D

 

26. Human traits, such as impulsivity and fearlessness, are as the characteristics of artists than those of criminals.  
                                                                        A                           
                                                       B       C               D

 

 

解答

 1-10

  D D D C缺

 B D A D B

 11-20

  D D B C A

   從缺    B

21-22

A D A A B

D D C D B

 

 

 

Zeus 發表在 痞客邦 留言(0) 人氣()

Test 8

 

1. Customers pay premium price for ______ products because the quality of the products is higher than that of similar products or they are heavily advertised.

 

(A) branded (B) branding (C) brandish (D) brandy

 

2. The higher the price, ______ for producers to produce and sell. So a rise in price increases the quantity supplied.

 

(A) the higher incentive (B) the incentive is higher (C) the higher the incentive (D) when the incentive higher

 

3. Sales promotion is intended to give a short boost ____ sales, such as offering discount vouchers, free gifts, and better value offers.

 

(A) for (B) to (C) with (D) at

 

4. The color of the packaging and its design need to reflect the image of the product _______ to give any information required by law.

 

(A) yet (B) as well as (C) but (D) neither

 

5. Sony is one of the companies throughout the world, which uses an ______  assembly line to produce digital cameras and varied video products.

 

(A) automation (B) automating (C) automated (D) automaton

 

6. The United Colors of Benetton, __________, has come to stand for up-market casual but smart clothes.

 

(A) which an Italian company

(B) is an Italia company

(C) that it is an Italian company

(D) an Italian company

 

7. Because the market cannot always prevent _______ their customers or supplying inadequate goods, governments have to step in to pass consumer protection laws.

(A) misled (B) mislead (C) misleaded (D) misleading

 

8. _____ a loan, where interest has to be paid, dividends to shareholders don’t have to paid if profits fall or the company makes a loss.

 

(A) Like (B) Unlike (C) Dislike (D) Likely

 

9. Balance sheets are used by other businesses and individuals to judge whether the business is safe enough _______in or to buy out.

 

(A) so that to invest (B) investing (C) to invest (D) and invests

 

10. The train across the English Channel connecting England and France speeds through the underwater tunnel ____ rates of 90 to 180 miles per hour.

 

(A) with (B) for (C) in (D) at

 

11. The Internet, a backbone for the so-called data superhighway, ________computer networks, cable TV, interactive phone services and other technologies.

 

(A) consists of (B) consisting of (C) was made of (D) made up of

 

12. _____ corporate America comes to terms with the antismoking fervor, more and more companies are regulating their use of tobacco in the workplace.

 

(A) As (B) Although (C) Whereas (D) Since

 

13. Research around the world points to a recipe for _______ - a low fat, high fiber diet with calories coming from the grains and legumes.

 

(A) be well (B) being good (C) well-being (D) good-being

 

14. Ethnocentrism is the view that one’s own culture is better than all the others; it is the way all people feel about themselves as _________outsiders.

 

(A) compared to (B) comparing to (C) are compared to (C) they compare to

 

15. Wherever you go, garbage, plastic bags, and bottles are thrown ________without regard.

 

(A) everywhere (B) nowhere (C) anywhere (D) wherever

 

16. Many things can happen and make the actual outcome different from ______ was budgeted.

(A) which (B) what (C) that (D) whether

 

17. In American football, two teams ______11 players attempt to score points by kicking goals or by putting the ball behind their opponents’ goal line.

 

(A) with (B) have (C) which has (D) to have

 

18. Sales turnover is equal to the average price of the product sold _____ the number sold.

 

(A) time (B) timing (C) times (D) timely

 

19. If the government ________ value added tax (VAT), then taxes on goods and services bought will be lower.

 

(A) will cut (B) cut (C) cuts (D) has cut

 

20. Mr. Buffalo insists that his secretary _______ weekly updates on each project and report them in the weekly meeting

 

(A) provides (B) to provide (C) provided (D) provide

 

21. The Market is like a vast voting machine with buyers and sellers being the market forces which determine price and quantity __________ in a market.

 

(A) buy and sell (B) bought and sold (C) buying and selling (D) bought and selling

 

22. The company makes traditional men’s shoes that will look good with a suit or __________.

 

(A) and more causal clothes

(B) with more casual clothes

(C) wear more causal clothes

(D) to wear clothes causally.

 

23. Sara’s ran up debts of $50,000 in the first year and it will ________business unless it increases sales to 4,000 personal computers in the second year.

 

(A) go out of (B) go off (C) go for (D) go through.

 

24. Helen prefers to wander around the open air market, _______ there are numerous stalls with traders selling everything from food to clothes.

 

(A) which (B) that (C) where (D) what

 

25. Primary industry, including fishing, mining, farming, and oil extraction, is the sector, where raw materials are extracted, _____ and cut down.

 

(A) growing (B) grew (D) grown (D) grow

 

26. Changes in _____ is being produced in the economy are bound to lead to changes in where and how people work.

 

(A) that (B) which (C) whether (D) what

 

27. The Manufacturing industry ________ over the past thirty years, causing the manufacturing employment as a proportion of total employment fell by 17 percent.

 

(A) was revolutionized

(B) has revolutionized

(C) revolutionized

(D) had revolutionized

 

28. Tesco, one of the largest supermarket chains in the UK, employing up to several hundred full time workers, ______ wealth and prosperity in the local community.

 

(A) creating (B) creates (C) create (D) have created

 

29. The size of population in a local area would affect businesses; _____ what happens to business could well affect the size of population.

 

(A) similarly (B) therefore (C) consequently (D) even though

 

30. The _______ of a city, such as roads, telephone networks, water, gas and electricity supply, has an important impact on local economical development.
(A)從缺 (B) infrastructure (B) environment (C) policy (D) construction

 

 

解答

 1-10

 A C A B A

 D D B C D

11-20

 A A C A A

 B A C C D

 21-30

 B B A C D

 D B B A B

 

 

 

Zeus 發表在 痞客邦 留言(0) 人氣()

Test 7

 

1. The name list of the new executive directors, ______ was announced today, would be very surprising and beyond people’s expectations

 

(A) when (B) whose (C) it (D)which

 

2. An attractive display is one thing that encourages customers _____ the products of the company.

 

 (A) to buy (B) buying (C) buy (D) bought

 

3. ______one gets, the more precious friendship becomes; for it affirms the contours of our existence.

 

 (A) As older (B) Being older (C) The older (D) As older 

 

4. An estimated five and a half billion tones of carbon ______ released into the atmosphere each year by burning of coal, oil, and synthetic fuels.

 

 (A) have been (B) have (C) has (D) has been

 

5. If a relaxed baby _______by loving, attentive parents, he might grow up to be an intelligent risk taker – an artist, or a scientist.

 

 (A) is raised (B) was raised (C) were raised (D) has been raised

 

6. Anyone who has experienced a gut-busting laugh knows ____ it interrupts tension.

 

 (A) how (B) what (C) which (D) when

 

7. At work or home, the basic tenet for humor is: take your responsibilities seriously, _____ don’t take yourself so seriously.

 

 (A) and (B) but (C) either (D) nor

 

8. There is some evident that _______ mentally active can slow age-related decline.

 

(A) keeps (B) keeping (C) kept (D) while keeping

 

9.Men tend to be seduced by technology; _____ they get into the faster-race-car syndrome, bragging about the speed of their microprocessors.

(A) but (B) while (C) for example (D) due to

 

10. The telephone is an extension of the human voice and the telescope and microscope are extensions of their ______.

 

(A) seeing (B) seen (C) sight (D) sought

 

11. Since their inception, the department has been troubled by high turnover.
               A                                                   B                   C      
                    D

 

12. The figure compares with a effective tax rate last year of 36 percent which resulted from write-offs.
                                        A    B                                                  
                                       C                      D

 

13. Steven Spielberg, one of the most successful Hollywood movie director, shows his creativity in the diversity of his films. 
                                                      A                                              B    
                        C                                    D

 

14. Despite products of the technological age –TV, videos,computer games – have taken many people away from reading books, the book industry is

             A                                                                                                                              B                                                        C

      still thriving.
          D

 

 

15. Increasingly, waste is being seen as a resource that the world can not longer afford to squander.
        A                                   B                                                                   C                                                         

 

 

16. If the service is not prompt, the customers would have gone to another restaurant.
                A                   B                                                 C                    D

 

 

17.The director asked Helen gives the customers a tour of the factory and a demonstration of the latest products.
                                                A                                  B                                             C                            D

 

 

18. The letter that Mr. Howard sent it from Taipei has a lot of typographical errors.
                     A                                 B                       C                       D

 

 

19. The staff are required to attend the company training sessions a week twice.
                   A                       B                                      C                
             D

 

20. Some people think it is good to censor books and the other believe that censorship goes against the basic right of freedom of expression.
                                                           A                              B                                                         C                                                                     D

 

 

21. Stockbrokers are people who buy and sell shares for their clients and charging fixed rates of commission for this service.
                                                 A                           B                                         C               D

 

 

22. Researchers have found that the healthiest diet is simple, inexpensive, and traditional fare that people abandon as they moving into affluence. 
                                                             A                                                                            B                                                                      
C

 

23. The items on this invoice were mislabeled and has incorrect stock identification number. 
                    A                                      B                    C                                     
 D

 

24. The visitors only has a twenty-dollar bill with her when she landed at Hartford airport. 
                                A             B                                                        
C     D

 

25. Due to poor planning and problems with local cash flew, most analysts predicted that the firm remained solvent for only a few months.
      A                                                                               B                                                                              C                                      D                        

 

 

26. The writings of Helen Keller serve as an inspiration not only for people with disabilities and for all people the world over.
         A                                                                B                                                        C            D

 

27. If e-mail had been invented before the telephone, it would be the most popular communications medium. 
                   A                                                                             B            
   C                              D

 

28. Modern children should be taught to deal direct with reality and to be proud and confident.                        

                                                                                   A              B                  C                              D

 

29. The director had her assistant picked up some related references for the proposal which will be submitted at thecoming quarterly meeting. 
                                                           A                       B                                                                                   C                                      
 D

 

30. As business has become increasingly international, the need for clearly-understood communication between members of different cultures are

                                                                                    A                                                         B                                                                                                                       C

even more crucial.
                D

 

 

解答

 1-10

 D A C A C

 A B B C C

 11-20

 A B B A C

 B A B D B

21-30

 C D C A B

 D B A A C

 

 

Zeus 發表在 痞客邦 留言(0) 人氣()

Test 6

1. Advertisers use all kinds of _____ to get your attention and your dollars.

 

(A) formula (B) process (C)strategies (D) arrangement

 

2. Those ______ expertise was surpassed only by their diligence.

 

(A) technician (B) technical (C) technique (D) technology

 

3. Please tell other staff who are addicted to tobacco _____ both in the hall way and in any working areas.

 

(A) don’t smoke (B) not smoking (C) aren’t smoking (D) not to smoke

 

4. The representatives are _____ going home and seeing their families after this negotiation.

 

(A) looking up to (B) looking for (C) looking out (D) looking forward to

 

5. Our company is thinking of _____ its operations into foreign markets.

 

(A) raising (B) inflating (C) enhancing (D) expanding

 

6. The secretary typed up the monthly report in time but it is_____ not correct.

 

(A) already (B) still (C) anyway (D) yet

 

7. The ______ are too strict for the subcontractors to meet

 

(A) specifications (B) specified (C) specific (D) specifying

 

8. If the marketing strategies are well applied, we _____ our profits by 20 percent.

(A) will have been increase

(B) would have increased

(C) will increase

(D) increase

 

9._______, friendships, etiquette, patience and protocol are integral parts of business.

 

(A) Socializing (B) Social (C) Sociable (D) Society

 

10. In some countries, business people have a very direct style, while in ______ they are much more subtle in style.

 

(A) others (B) the others (C) another (D) each

 

11. It has recently been proved_________  the laser can be used to perform bloodless, painless surgery.

 

(A) where (B) until (C) while (D)that

 

12. All the companies cannot afford to make costly advertising mistakes _____ they want to be competitive and profitable.

 

(A) if (B) why (C) how (D) that

 

13. When you eat _______, you get the vitamins you need from the foods you eat.

 

(A) a diet of well balance

(B) to balance a diet

(C) a diet is well balance

(D) a well-balanced diet

 

14. We only give discounts of more than 20% to customers with whom we have ______a good, long-term trading relationship.

 

(A) set off (B) set for (D) set off (D) set up

 

15. Hospitals are competing for shrinking market share; ________ they are attempting to discover the most cost-effective and highest quality care.

 

(A) therefore (B) also (C) besides (D) furthermore

 

16. Football and baseball ______ currently played in the United States are basic modifications of games that originated in England

 

(A) which (B) are (C) as(D) that are

 

17. Not every person who is indicted ____________.

 

(A) of guilt (B) is guilty (C) to be guilty (D) guilty

 

18. Understanding and heeding cultural _______ is one of the most significant aspects of achieving successful in any international business enterprise.

 

(A) varies  (B) differs (C) variables  (D) different

 

19. Educators reminded that the subtext of many advertisements can be_______; some promoting sexism and racism and other encouraging ethnic stereotyping.

 

(A) agreeable (B) attractive (C) controversial (D) diversified

 

20. We cannot process the order _____ we get a copy of the purchase order.

 

(A) because (B) that (C) until (D) when

 

21. Please use the _____ envelop for your reply.

 

(A) is enclosed (B) enclose (C) enclosing (D) enclosed

 

22. The customer service representatives ______ at the receptionist desk for product introduction.

 

(A) always are available

(B) are always available

(C) are available always

(D) being always available

 

23.The publisher expects circulation ______ in the coming new year.

 

(A) to ascend (B) to increase (C) to escalate (D) to raise

 

24. Using the checklist is an ____ way to make plans.

 

(A) effective (B) effected (B) affective (D) affective

 

25. The ______ about out recycling plans will reassure consumers.

 

(A) public (B) publish (C) publication (D) publicity

 

26. The captain requested that all passengers ____ emergency procedures.

 

(A) reviewed (B) reviews (C) reviewing (D) review

 

27. Scholars and experts on economics suggest the government ______ a research institute of economic development

 

(A) organizing (B) organize (C) organized (D) organizes

 

28. The Ford-model car has always been in good condition and seldom been brought in for _______.

 

(A) repairs(B) despair (C) impairs (D) comparisons

 

29. Our latest advertising packages include brochures _______ videos.

(A) or (B) either (C) neither (D)and

 

30. Mr. Wang’s secretary will_____ the messages if he has not time to answer the phones.

 

(A) make (B) do (C) take (D) leave

 

 

 

解答

 01-10

 C B D D D

 B A C D A

 11-20

 D A D D A 

 D B C C C

 21-30

 D B D A C

 D B A D C

 

 

Zeus 發表在 痞客邦 留言(0) 人氣()

Test 5

 

1. The appeals court, _______, has agreed that the company did not violate antitrust laws.

 

(A) it decided to review the case

(B) which it decided to review the case

(C) decided to review the case

(D) which decided to review the case

 

2.The bids on ________ the complex have been submitted by the construction companies

 

(A) putting off (B) putting up (C) putting on (D) putting through

 

3. The ____ market has declined in many parts of the city since the end of last year.

 

(A) house (B) homing (C) homes (D) housing

 

4. If our candidates _____ elected, we will have a very strong influence in deciding the direction of the future development of our company.

 

(A) will be (B) are (C) were (D) have been

 

5. _______ you drive into Taipei or arrive via some other form of transportation, you’ll find the city easily accessible.

 

(A) If (B) Although (C) What (D) Where

 

6. The researcher helped the department store ______ a thorough job of polling potential customers.

 

(A) do (B) done (C) to do (D) has done

 

7. Several difficulties have ____ in our negotiations over the new contract.

 

(A) risen (B) raised (C) arisen (D) aroused

 

8. According to the ____ of the invoice, settlement of this account was to be made within ninety days.

 

(A) codes (B) items (C) words (D) terms

 

9. Because home-based business owners were unable to compete with the production capabilities of the new factories, they were forced ______ business and became employees in the factories.

 

 (A) out of (B) on (C) in (D) for

 

10. The city’s historic buildings has been shaping a nation and its many fine art museums and great educational institutions are known worldwide.

              A                                                    B                               C                                                                                                                       D

 

11. As arranging, the video scanners will be available for collection from our warehouse on 3 June.

             A                                                           B                     C                                        D

 

12. Your intelligent and energy will help you solve the problems you encounter, use your strength wisely.

                     A                                                 B                                       C                                             D

 

13. Blazing autumnal foliage, sparkling ski slopes, acres of spring blossoms, a wealth of summer music, and country fairs are all part of New

                                                                                                               A                          B                                                             C

       Hampshire’s delight.

                                              D

14. We look forward to hearing of the safe arrival of the replacements and to doing farther business with you.

                                        A                           B                                                      C        D

15. Before the company remodels its office, a questionnaire was designed for the employees to express their opinions on factors that effect their

                                                                                                               A                                         B                                 C                  D

       working environment

 

16. The committee members decided to stop to interviewing those applicants who were not willing to be relocated.

                                                                      A                                  B                                          C                                    D

 

17. The Board of Directors recently suggested the office workers that all smokers went outside to smoke.

                                                 A                                                       B                          C                   D

 

18. There is a rumor that school teachers are discussing the establishment of an union.

         A                         B                                  C                                                     D

 

19. The editors of the monthly newsletter, which contains literary articles and cultural events, has decided to accept commercial advertisements.

                                         A                                                            B                                             C                                           D

20. In order to select the staff members who best deserves this year’s trip to Thailand, all the shareholders are requested to read the enclosed

                                                                             A                                                                                                      B                                  C

    profiles on the candidates beforehand.

                                                         D

 

21. The disks which contained the information you require was accidentally destroyed.

                                    A                                                 B        C          D

 

22. Cross your legs in a relaxed manner in front of an American could be regarded as rather effeminate.

             A                               B                                                                                       C                   D

 

23. Early settlers made quilts primarily for warm and economy with leftover scraps of materials from the household.

                                                     A                B                                           C                                                      D

 

24. Your paychecks can be deposited directly into your account, or you can pick it up at the Accounting office.

                                                               A                            B                        C                       D

25. Mostly three-quarters of the nation’s lobsters are caught off the coast of Maine.

           A                B                                                          C                      D

26. Today, the stock market surged to it’s third record high and gained two hundred points for the week.

                                                                     A           B                  C                   D

27. According to the survey, allow deductible contributions would encourage about one million households to invest.

                    A                               B                                                                                           C                                   D

28. Cape Cod is curiously shaped; some people see it as a flexed arm and the other see it as perhaps a fishhook.

                                            A                                                    B                                  C                                         D

29. If we do not receive your payment by the end of this month, we shall have no alternative and to place the matter in the hands of our solicitors.

                                                                 A                                                                 B              C                                           D

30. By using a word processor linked to a personal computer and connected to the company’s computer, it is possible to exchange the information

            A                                                                                                                                                                                    B

required to do a wide variety of office job at home.
                  C                                       D

 

 

解答

1-10

 D B D B A

 A C D A B

11-20

 A A D D D

 B C D C A

21-30

 A A B D A

 C B C C D

 

Zeus 發表在 痞客邦 留言(0) 人氣()

Test 4

 

1. The sales division reported a 35 percent increase ____ the last sale period.

 

(A) during (B) with (C) at (D) as
 

2. By the end of this year, economic and political situations _______ greatly.

 

(A) will be changed (B) will have changed (C) changes (D) changed.

 

3.___________ time to submit a bid.

 

(A) Still there is (B) There is still (C) There still is (D) It is still.

 

4. After attending a seminar on Communicating Effectively at Work, Mr. Brown picked up some interesting pointers, ______made contact with several people who could possibly be potential clients

 

(A) as well as (B) also (C) either (D) or

 

5. Each unit has been encased in foam rubber and then packed in a cardboard box to protect it from rough handling in ______,

 

(A) transfer (B) transit (C) transition (D) transportation

 

6. All applicants ______to be considered for the grant must submit a letter describing their enterprise and why they feel they should be given the grant.

 

(A) wishing (B) wish (C) have wished (D) will wish

 

7. A Bill of Lading is a receipt given by the shipping company _____that the goods have been loaded on board ship.

 

(A) to say (B) to tell (C) to speak (D) to confirm

 

8. The _____ total working time to complete the project is six weeks, but it might take longer or shorter time to make it done.

 

(A) anticipated (B) reexamined (C) realized (D) practicable

 

9. Baltic Exchange has its origin in the 17th century coffee houses, ____merchants would meet to drink coffee, read newspapers, and transact business.

 

(A) which (B) that (C) where (D) whose

 

10. The future development of our company is based on _____ we can provide.

 

(A) what services (B) how services (C) whether services (D) that services

 

11. The Central Downtown _______ as the costliest place in the city to do business.

 

(A) often is referred (B) is often referred (C) is referred often to (D) is often referred to

 

12. Since the early part of this year, our citizens _______ with the policies and services of our government.

 

(A) are not satisfied (B) have not satisfied (C) have not been satisfied (D) have been not satisfying

 

13. The newly appointed Minister of Education _______ critics and answer the questions submitted by the journalists.

 

(A) stands in (B) stands over (C) stands by (D) stands up to 

 

14. The clients would like the invoices ____ directly to their offices.

 

(A) fax (B) faxing (C) faxed (D) be faxed

 

15. Mrs. Roarke, who could not give the presentation on marketing techniques scheduled on March 6th, called to offer an _______time for her presentation.  

 

(A) advantage (B) appealing (C) allusive (D) alternative

 

16. FAX, the abbreviation for facsimile machine, was invented in 1901, but until recently _______mainly for transmitting newspaper photographs and weather maps.

 

(A) which was used (B) were used (C) using (D) used

 

17. The mayor wants the meeting _______ immediately.

 

 (A) to arrange (B) arranged (C) will arrange (D) arranging

 

18. We thought the project would be a great success; _______ it failed to attract customers’ interest.

 (A) thus (B) yet (C) besides (D) accordingly

 

19. The ______ department is responsible for recruiting employees and keeps a file on each employee.

 

(A) person (B) personal (C) personnel (D) personable

 

20. As soon as the budget ________ approved, the project will be start immediately.

 

(A) is (B) has been (C) was (D) will be

 

21. A strike will lead _____ a settlement that will be ultimately mandated by the Legislative Yuan.

 

(A) from (B) to (C) at (D) with

 

22. A charity foundation was _______ to help the homeless and the unemployed.

 

(A) caught on (B) went over (D) filled out (D) set up

 

23. I prefer to buy things with high quality ________they are not currently in vogue.

 

(A) even if (B) and (C) in that (D) whereas

 

24. These statistics only show the situation of local commercial operations; ______ the international trade is not included.

 

(A) moreover (B) consequently (C) on the whole (D) furthermore

 

25. The firm offers new employees ______ care and life insurance benefits.

 

(A) healthy (B) health (C) healthful (D) healthfulness

 

26. The flight will arrive in Paris ____ twenty minutes.

 

(A) at (B) for (C) in (D) until

 

27. Currency exchange losses from the transactions need to be assessed _______.

 

(A) annually (B) rarely (C) always (D) more or less

 

28. Ms. Shirley is aggressive and wants to _____ more responsibilities.

 

(A) take in (B) take on (C) get to (D) get on

 

29. There have been no grievances _____ complaints reported in the last six months.

 

(A) with (B) also (C) but (D) or

 

30. The latest report provides analysts with ______ confidence in solid economic growth for the first half of the year.

 

(A) renewing (B) renewal (C) renewed (D) renew

 



解答

1-10

   A B B A D

 A D A C A

11-20

D C D C D

 D B B C A

21-30

B D A B B

 C A B D C


 

Zeus 發表在 痞客邦 留言(0) 人氣()

Test 3
 

1. Our department did not reach its monthly quota ____ we worked a lot of overtime.
 

(A) because of (B) despite (C) although (D) but
 

2. The report ____ the process of solving the mechanical problems is on the shelf.
 

(A) explained (B) explains (C) who explaining (D) that explains
 

3. The person submitting the complaint must include a detailed description of ___________.
 

(A) what occurred (B) it occurred (C) something occurred (D) which was occurred
 

4. A Facsimile machine is a machine ________of a document via the international telephone network.

(A) which it transmits an image
(B) transmitted an image
(C) that transmits an image
(D) what transmits an image

 

5. Along the Connecticut River, Hartford is an attractive, thriving mixture of the old but new.
      A                                                               B                         C                                 D

 

6. The rapid advances in technology make a branch network less important as it used to be as a base for offering services to the public.
                                                               A                                                         B          C                   D

 

7. Progress must be careful controlled so that the company will not grow too fast. 
         A                         B                             C                                                     D

 

8. The traveling guide advises us to visit Italy in winter, where the prices are lower.
             A                                          B                                  C                              D

 

9. If you have just recently purchased a car, or run a new company, you might consider to take advantage of the low interest rates of the City bank.
                                         A                                  B                                                                    C                                              D

 

10. The products requiring should be described in detail in order to expedite the shipment.
   A                         B                                                  C                      D

 

11. After being asked his advice, Mr. Harmoui recommended to  have the proposal proofread by a lawyer.
              A                  B                                                                 C
                                                   D

 

12. Some people view gambling as a harmless form of fun that makes life more interesting; the other think of it as an evil enterprise that takes money

                                                                                                                                         A                                                  B

       from the poor.
                C

 

13. With one in two marriages ending in divorce, it is an increase in single-parent homes.
                  A                                B                                                              C

 

14. The commodities market is a telephone market, taking place in the dealing rooms of major banks, where currencies are bought and sell all over

                               A                                                                 B                                                                               C                                                   D

      the world.

 

15. Because the pace of technological change and the intensity of competition, it is likely that the number of firms dealing in shares will become

           A                                                                                    B                                              C                                                        D

    fewer and larger.
 

16. The position will be offered to the person who have better qualifications than the other candidates.
                                           A                                   B               
              C                                         D

 

17. Attempts to invade privacy is illegal and violate standards of conduct.      

                              A                     B                         C            D

 

18. We look forward to hearing that your order has arrived safely  and to do business with you in the future.
                                          A                                                        B     
              C                    D

 

19. Considering by many to be the birthplace of American democracy, Boston attracts visitors from all over the world.
             A                          B                   C                            
                                                  D

 

20. The passengers, which were frustrated by the delays, were crowded in front of the ticket counter to change their flights.
                                      A                                           B                                         C                                                                     D

 

21.The boss was very angry because the letter that the secretary taped it had many careless mistakes.
                                                  A                         B                                         C                     D

 

22. Last month the interest rate that we paid on our bank loan raised from 12% to 15%.
                                  A                                    B                          
      C                    D

 

23. Both the shipping charges or the handling charges are included in the price.          

                            A                      B              C                        D
 

24. The development of steam-powered machinery in the eighteenth century resulted from the rapid growth of factories.
                                                   A                                    
           B                                  C                             D

 

25. An exploration of medical studies has produced a flood of  information on diverse human diet.
               A                                                      B                 C                                                 
      D

 

 

解答

1-5

C D A C D

6-15

B B C C B 

 C B C C A 

16-25

B B C A A

 C C B C D

 

 

單字補充
 

rise 起身 升起 (太陽、煙霧) 上漲(河水、價位)晉升

arise 出現 發生(困難、爭議、需求)

arouse 喚醒 引起 激起(suspicion, sympathy, laughter)
 

Zeus 發表在 痞客邦 留言(0) 人氣()

Test 2

 

1. When the contracts are ready, have them _____ to the purchasers.
 

(A) send (B) sent (C) be sent (D) been sent
 

2. The variety of insurance benefits ____ very broad under this policy.
 

(A)are (B) is (C) be (D) has been

 

3. _____ the figures had been checked twice, the supervisor found a mistake.
 

(A) Unless (B) However (C) Since (D) Even though
 

4. Television has been around in the Untied States for half a century, and it is still a controversial topic, which ______strong and contradictory opinions.
 

(A) rises (B) arouses (C) arises (D) rose
 

5. Instead of working out in the office, brainworkers can now “telecommute” from home________ physically commute to the office.
 

(A) as well as (B) rather than (C) but (D) better than
 

6. Mr. Delta was responsible for collecting and _____ the related data for the proposal.
 

(A) organization (B) organized (C) organize (D) organizing
 

7. A four-month investigation made it clear that America today is well into the kind of troubling world ______the most basic principles of privacy are under attack.
 

(A) when (B) which (C) in which (D) wherever
 

8. The surgeon general’s report ______that smokers create health risks for nearby nonsmokers has encouraged companies to promote smoke-free work environments.
 

(A) asserting (B) asserts (C) asserted (D) has asserted
 

9. More and more companies that have imposed ______on smoking are attempting to help their employees kick the habit.
 

(A) restrictions (B) retention (C) restoration (D) reward
 

10. The Central Bank exercises general authority and _____over the banking and financial system in Taiwan.
 

(A) supervises (B) supervising (C) supervision (D) superbly
 

11. You may rely on us to _______the instruction in the Documentary Credit in full.  
 

(A) carry through (B) carry on (C) carry out (D) carry away.
 

12. We have already contacted our forwarding agent to make arrangements for the ________of the goods, and we assure that we can meet your delivery deadline of 31st June.
 

(A) transportation (B) transformation (C) translate (D) transition
 

13. When insurance is taken out, a proposal from is completed, which gives details of ______, for how long and the nature of the risk.
 

(A) what it is insured (B) what is insured (C) that it is insured (D) which is insured.
 

14. It is ______to seek advice from an insurance broker about the many different policies available from insurance companies.
 

(A) advisable (B) advised (C) advice (D) advising
 

15. ______ for the perfect location and assessing the needs of their company, the members of the executive committee took a long time before making their decision.
 

(A) Look (B) Looking (C) Looked (D) Because look
 

16. To prevent errors caused by using the new coding system, we should ______ before beginning to write codes.

(A) find it over (B) take it over (C) look it over (D) look it up

 

17. Please pack the boxes in wooden cases, five boxes per case which must have fireproof ____ waterproof linings.
 

(A) with (B) so (C) and (D) but
 

18. Many employees in this company cannot use computers; _____ in-service training is required.
 

(A) on the whole (B) besides (C) consequently (D) and
 

19. If there _____ better communication, the couple would not divorce.
 

(A) was (B) is (C) will be (D) were
 

20. _____ you do not receive the original invoice, we will enclose a copy with this letter.

(A) While (B) Whether (C) In case (D) As long as

 

21. Money can be considered as a ______and it is traded in the foreign exchange market.
 

(A) facility (B) commodity (C) loan (D) credit
 

22. In order to avoid making mistakes, the manage have his colleagues _____ the statistics.
 

(A) checked (B) to check (C) check (D) who check

 

23. Your _____of the thirtieth of May has come to our attention, and we wish to acknowledge your receipt immediately.
 

(A) request (B) requirement (C) demand (D) denial
 

24. The prices of shares change from day to day ______supply and demand and the rise or fall of the market.
 

(A) according upon (B) in accordance with (B) match with (D) are based on
 

25. The habit of traveling long distances to work _______for the past decades due to the availability of mass-produced reasonably-priced cars.
 

(A) is (B) was (C) has grown (D) had been
 

26. Quilting requires a _______investment of time, which is why prices are often high.
 

(A) considerate (B) considering (C) considerable (D) considered
 

27.______a country’s food become rich, the diseases of poverty (malnutrition, infectious disease) are replaced by the those of civilization (arteriosclerosis, obesity)
 

(A) Whereas (B) As (C) Even though (D) Unless
 

28. With this change in people’s working habits _______ the new problem of filling their free hours with recreational activities.
 

(A) come (B) comes (C) have come (D) coming
 

29. Many people _____debt because they overuse their credit cards, and they are charged at 12 to 20 percent interest on the money they borrow on their cards.
 

(A) get over (B) get through (C) go into (D) get off
 

30. Dr. McClinton was the first woman to win the Nobel Prize for medicine alone and the third woman _____any unshared Nobel science prize.
 

(A) to win (B) winning (C) who wins (D) she won
 

31. The National Recycling Coalition recently _____Congress to pass tax incentives that would encourage development of new applications for waste materials.
 

(A) calls up (B) call over (C) call for (D) called on
 

32. A trusting friend can call us back to earth and remind us _____ the universality of failure and sufferings.
 

(A) from (B) of (C) on (D) with
 

33. If the earth ______ warmer each year, so called greenhouse effect, the earth would become a dead planet like Venus
 

(A) will get (C) would get (C) gets (D) were to get
 

34. The increased carbon dioxide is probably not a threat to health, _____ it profoundly affects the way the earth is heated by the sun.
 

(A) nevertheless (B) thus (C) and (D) but
 

35. It has recently been proved_________ lasers can be used to perform bloodless, painless surgery.
 

(A) where(B) until (C) while(D)that

 

解答

1-10

C B D C B   

 D C A A A

11-20

C A B A B

 C A C D C

21-30

B C A B B

 C A B C A

31-35

C B D D D 

Zeus 發表在 痞客邦 留言(0) 人氣()

Test 1

 

1. A free gift will be provided with every purchase of NT 500 or more for a _____ period of time. 

(A) limit (B) limited (C) limiting (D) limitation

 

2. The Youth Club has arranged for card members to receive one thousand dollars of _____ life insurance.

(A) compliant (B) complimentary (C) comprehensible (D) compromise

 

3. Any person who is _____ in applying the position of professional secretary should send his or her resume and three references to the personnel
   director. 

(A) interesting (B) interest (C) interested (D) interestingly

 

4. Mr. Lee was very concerned _____ the upcoming board of directors meeting. 

(A) to (B) about (C) at (D) on

 

5. There is no ______ made on weekends because the hotel is almost occupied.
 

(A) delay (B) violation (C) appointments (D) reservations

 

6. Employees are ____ that due to new government regulations, there is no smoking in the working places.

(A) reminded (B) respected (C) reacted (D) remembered.

 

7. On international shipments, all duties and taxes are paid by the ________.   

(A) recipient (B) receiving (C) receipt (D) receptive

 

8. Because of the _____ weather conditions, Hawaii has an advantage in the production of fruits and the promotion of touring business.

(A) favorite (B) favor (C) favorable (D) favorably

 

9. I has been predicted that an _______ depreciating Taiwan dollars will stimulate exporting business in Taiwan.

(A) increased (B) increasing (C) increase (D) increasingly

 

10. Please present the VIP card, when ______ charges to your room account or making a reservation for sporting facilities.

 

(A) signature (B) singing     (C) signed (D) sign

 

11. The firm is not liable for the damage resulting from some conditions ____ its control

(A) beyond (B) above (C) more than (D) over

 

12. Initial _____ concerning the merge of the two banks will be held next Monday at the International Trade Center.

(A) negotiations (B) dedications (C) propositions (D) announcements.

 

13. The corporation, which underwent a major restructuring a couple years ago, ______ steadily for three years.
 

(A) grew (B) had grown (C)has been growing (D) to grow

 

14. ______ have the marketing competitions been more complicated and subject to change.

(A) Mostly (B) Negatively (C) Totally (D) Rarely.

 

15. All full-time employees are _____ to participate in the health plan, which takes effect on the first day of this month.

(A) eligible (B) elevated (C) elicited (D) elected

 

16. _______ terms of quality and service, the Excellence company surpasses the competition.

 

(A) In        (B) From        (C) By    (D) With

 

17. The construction of the Rapid Transit System will help the city ________. 

(A) diversify (B) clarify (V) verified (D) modified

 

18. As the airplane is taking off, passengers should return their seat backs to full upright and _______their seat belts.
 

(A) lock (B) fastened (C) fasten(D) secure

 

19. Making appropriate arrangement for audiovisual equipment in advance is _______ recommended for all seminars.
 

(A)sternly (B) strikingly (C) stringently (D) strongly

 

20. Be alert –use your eyes, ears, and intuitions when negotiating, and establish good ______ with the other negotiating team.
 

(A) affection (B)compassion (C) rapport (D) friendship

 

21. Interviewees are given the company brochure to read ____ they are waiting for their interviews.
 

(A) during (B) while (C) nevertheless (D) whereas

 

22. Stocks usually benefit ____ the January effect, which causes the price of these stocks to rise between November and January.

(A) of (B) in (C) for (D) from

 

23. While working in the trade exhibition, every one is required to wear his or her identification badge at all times so that he or she is _______ as a company employee.
 

(A) recognize (B) recognizing (C) recognizable (D) recognizably

 

24. Currently, there is an excellent ______ for our society to boost its economic development.
 

(A) popularity (B) regularity (C) opportunity (D) liability

 

25. It is required to ______ all relevant data to the association ten days prior to the beginning of the conference.

(A) emerge (B) substantiate (C) adapt (D) submit

 

26. It is ______ that Ms. Jin wishes to handle all transactions by herself but it might be better if several people shared the responsibility.

(A) admire (B) admiring (C)admirable (D) admired

 

27. Passengers are reminded to allow for ____ travel time to the airport during the (A) rush hours.

(A) addition (B) additive (C) additional (D) adding

 

28. ____ higher cost this year, attendance at the management training seminar remains above average.

(A) Even though (B) Nevertheless (C) Despite (D) Because of

 

29. Any unsatisfied item must be returned within 30 days and ____ by the original receipt from this store.

(A) altered (B) adjusted (C) accompanied (D) aggregated

 

30. The committee members have approached the proposal with a good deal of ______ because some of the ideas put forward are very
     controversial.

(A) cautious (B) caution (C) cautioned (D) cautiously

 

31. The salesperson has been negotiating the price and come to his best offer; ________, his client is still not satisfied and tired to bring down the price.
 

(A) however(B) although (C) therefore (D) whereas

 

32. During this fiscal year, the Twin Head Co. was _______and its production raised from 700 cars a month to 1000 cars a month.
 

(A) widened (B) strengthened (C) amplified (D) enlarged

 

33. The company is recruiting a regional manager with a starting salary of NT$ 40.000 plus bonus, a prestige car ____ other fringe benefits.
 

(A) or (B) and (C) but (D) nor

 

34. His credit card was canceled because his monthly payment is several months _________.
 

(A) due (B) overdue.(C) update (D) out of date

 

35. Northwest Flight 887 from Detroit to Taipei with an intermediate stop in Japan will ________ in 20 minutes.
 

(A) depart (B) abandon (C) hasten (D) advent

 

36. In addition to its favorable location, the hotel has many _______, including a large indoor swimming pool, sauna, whirlpool, and exercise rooms in a wonderfully exotic décor.
 

(A) amenities (B)appliance (C) appendices (D) approaches

 

37 An auction for the estate of Dunhill Area has been _____ Saturday, June 20, at 12:00 noon.
 

(A) caught up with (B) called for (C) planned for (D) set for

 

38. In order to modernize the communications system, our system will be upgraded to handle ten extensions lines, with a built-in answering machine, conference calls and _______features.
 

(A) dialing-speed (B) dial-speed (C) speed-dialing (D) speed-dial.

 

39. About 500 _______technicians are on strike because more than 8,000 assembly-line workers were laid off.
 

(A) maintenance(B) maintained (C) maintaining (D) sustaining.

 

40. To add to your account, send a check or money ____ to Smith Funds along with the additional investments form.
 

(A) paid (B) pay (C) payable (D) payment
 

41. When boarding an airplane, passengers must present their boarding passes to the designate agent at the airport gate.
                   A                                                                                       B                                C                 D

 

42. If you need to find a list of books available through the library system, consultation the computer terminal near the reference room on the

                                        A                       B                                                              C

second floor
   D                                  

 

43. Special care has been made to see if the merchandise has been packed according to the customers’ instruction.
                                   A                     B                                                                 C                                               D

 

44. The national panel of experts exported that the related authority should continue to investigate the safety standardon a regular base.
                                                                                     A                                                        B                            C                                     D

 

45. For each workshop, participator must register and pay prior  to the data which the conference begins. 
  A                                                                B                       C   
                               D

 

46. Except we receive a definite commitment by the end of the month, we will be forced to reconsider our original proposal.
     A                                         B                   C                                                                                         D

 

47. The Hilton Hotel has a courtesy bus which runs every one hour both to from the airport. 
                               A                                   B              C     
                               D

 

48. One project, proposed by the Ministry of Planning and Investment, it calls for building a new terminal that can process at least eight million

                                A                                                                                          B                                                                C                          D

    passengers a year.
                           

49. According to the company policy, all computer disks, which refer to any computer-data medium, must be scanned immediately upon enter to

                                                                                                              A                                             B                                 C                           D

      the building. 
   
                        

50. After an account at Ford’s Mutual Funds has been established, one may purchase additional share by electronic transfer
                    A                                                                      B                                                       C                       D

 

51. There are many aspects of the position which are very appealing to me; but I believer it is in our mutual best interest that I decline your kind

                                                                                                      A                    B                     C                                              D

     offer.
     

52. The consulting company suggests relocating our headquarter at the suburbs to obtain more spacious space.
                                                                      A                                    B                          C                              D

 

53. In the event for any changes need to be made in the product, please inform our office immediately.
                             A                                                                   B          
               C                                 D

 

54. We appreciate your interest in our programs and look forward to hear from you soon.
                      A                  B                                                                           C                 D

 

55. All the transactions in this bank was handled quickly and efficiently by well-trained tellers. 
         A                                                          B                          
         C                     D

 

56. It is recommended that you follow the format show in the sample when preparing an advertisement to be displayed in the magazines.
                     A                                                              B                                                                  C                            D

 

57. Vivid International has decided to expand its loan to make up with a decline in investment returns.                 

                                                                           A                       B               C                                  D

 

58. Ms. Chen wrote to the director to complain about the poor services she received during she stay at the hotel. 
                            A                                          B                         
                      C                           D

 

59. The Howard family were taken surprise when they were told the building they were living in has a severe structural flaw.
                                          A                    B                                      
                                                   C                                            D

 

60. Mandy’s Café was forced to closed because its inadequate ventilation, improper food storage, and operating without a health department

                                                      A              B                                                                                                       C

permit.
     D

 

 

Zeus 發表在 痞客邦 留言(0) 人氣()